Fråga:
Kan man generera motsvarande potential i kvantmekanik, givet ett visst energispektrum?
Revo
2011-08-13 18:21:18 UTC
view on stackexchange narkive permalink

Ett typiskt problem i kvantmekanik är att beräkna det spektrum som motsvarar en given potential.

  1. Finns det en en till en överensstämmelse mellan potentialen och dess spektrum?
  2. Finns det ett systematiskt sätt att beräkna motsvarande potential om svaret på föregående fråga är ja, med tanke på spektrumet?
[Mer om MO.SE] (https://mathoverflow.net/search?q=shape+drum).
Sju svar:
David Bar Moshe
2011-08-13 20:33:51 UTC
view on stackexchange narkive permalink

I allmänhet är svaret nej. Denna typ av inversa problem kallas ibland för: "Kan man höra formen på en trumma". En omfattande redogörelse av Beals och Greiner ( Anal. Appl. 7 , 131 (2009); eprint) diskuterar olika problem av denna typ. Trots det faktum att man kan få mycket geometrisk och topologisk information från spektrumet eller till och med dess asymptotiska beteende, är denna information inte komplett även för system så enkla som kvantmekanik längs ett begränsat intervall.

För ytterligare detaljer, se Apeiron 9 nr. 3, 20 (2002), eller även Phys. Rev. A 40 , 6185 (1989), Phys. Rev. A 82 , 022121 (2010), eller Phys. Rev. A 55 , 2580 (1997).

För en mer experimentell vy kan du faktiskt ha partikel-i-en-låda problem med olika formade lådor i två dimensioner som har samma spektra; detta följer direkt från Gordon-Webb isospektrala trummor ( Am. Sci. 84 nr. 1, 46 (1996); jstor), och det implementerades av Manoharan-labbet i Stanford ( Science 319 , 782 (2008); arXiv : 0803.2328), till slående effekt:

Ron Maimon
2011-08-14 01:05:00 UTC
view on stackexchange narkive permalink

Harmonic oscillator har samma spektrum som en svagare harmonic oscillator med en hård vägg vid x = 0.

LATER EDIT: Jag ser att jag måste vara mer tydlig --- potentialerna

  • $ V (x) = 2x ^ 2 - 2 $
  • $ (x>0) $ $ V (x) = x ^ 2 - 3 $ och $ (x<0) $ $ V (x) = \ infty $

har exakt samma spektrum.

Men i så fall får du bara halva spektrumet av den vanliga harmoniska oscillatorn, eller hur?
@Revo är korrekt. Till exempel är energispektrumet för den fullständiga harmoniska oscillatorn (1,3,5,7, ...) alla gånger $ \ hbar \ omega_ {full} / 2 $ och för den halva harmoniska oscillatorn är (3,7,11 , 15, ...) gånger $ \ hbar \ omega_ {half} / 2 $. Bara genom att ändra $ \ omega_ {half} $ med något oberoende av $ n $ kan du inte få villkoren att matcha (t.ex. för $ n = 0 $; du måste låta $ 3 \ omega_ {full} = \ omega_ {half} $, för $ n = 1 $ let $ (7/3) \ omega_ {full} = \ omega_ {half} $, för $ n = 2 $ let $ (11/5) \ omega_ { full} = \ omega_ {half} $).
@dr jimbob Det är sant, eftersom de fullständiga harmoniska oscillatorlösningarna oscillerar mellan jämnt och udda, kommer hälften av spektrumet inte att uppfylla gränsförhållandena vid ursprunget för den harmoniska oscillatorpotentialen med en hård vägg vid x = 0, nämligen vågfunktionen måste försvinna där.
Det beror på om du anser att en konstant energiförskjutning ingår i spektrumet. Med andra ord, om du ignorerar marktillståndsenergin och bara är uppmärksam på skillnaderna mellan energinivåer, som är mätbara, verkar de två systemen vara identiska för $ 2 \ omega_ \ text {half} = \ omega_ \ text { full} $.
@David Men spektra är relaterade, inte identiska.
@Revo: under de förhållanden som jag angav (dvs. ignorerar en konstant energiförskjutning) är de identiska. Hela SHO har energinivåer $ E_ {n, \ text {full}} = E_ {0, \ text {full}} + n \ hbar \ omega_ \ text {full} $, och halva SHO har energinivåer $ E_ {n, \ text {half}} = E_ {0, \ text {half}} + 2n \ hbar \ omega_ \ text {half} $. Energinivåskillnaderna är $ \ Delta E_ {mn, \ text {full}} = (nm) \ hbar \ omega_ \ text {full} $ och $ \ Delta E_ {mn, \ text {halv}} = 2 (nm ) \ hbar \ omega_ \ text {half} $, som är identiska om du ställer in $ 2 \ omega_ \ text {half} = \ omega_ \ text {full} $.
Jag började lägga upp åsikten att David har rätt; men jag börjar tro att han har fel, av en bisarr anledning. Spektra skulle verkligen vara identiska om vi talar om det spektrum som du kan mäta experimentellt. Inget experiment mäter det absoluta värdet på marktillståndsfrekvensen; bara skillnaderna. Men det finns ett problem: halvpotentialen inkluderar endast vågformer med udda paritet, och när de placeras i superposition med varandra kommer de att stråla mycket svagt, om alls. Det är inte helt uppenbart för mig att det till och med kommer att finnas ett mätbart spektrum.
@Everybody: du kan lägga till en konstant till en potential. Det är fortfarande en potential. Vad är den här diskussionen?
@Marty--- frågan är inte fysik, det är matematik --- kan du hitta Schrodinger-operatörer med identiskt spektrum. Detta är ett trivialt exempel. Mer triviala exempel är vilken potential som helst och dess översättningar och reflektioner, och ett mindre trivialt exempel är vilken potential som helst och dess supersymmetriska konjugat, när marktillståndet inte bryter supersymmetri.
Med andra ord säger Ron Maimons svar i huvudsak att om vi jämför hela SHO $ H = p ^ 2 / 2m + m (x \ omega) ^ 2/2 $ med halva SHO $ H = p ^ 2 / 2m + m (x \ omega / 2) ^ 2/2 - (\ hbar / 2) (\ omega / 2) $, $ x \ geq 0 $, med halva frekvensen $ \ omega / 2 $, och med nollpunkt energi $ (\ hbar / 2) (\ omega / 2) $ subtraherad, då är de två spektra identiska $ E = \ hbar (\ omega / 2) [(2n + 1) +1/2] - (\ hbar / 2) (\ omega / 2) = \ hbar \ omega (n + 1/2) $.
Qmechanic
2011-10-29 15:24:01 UTC
view on stackexchange narkive permalink

I det här svaret kommer vi bara att överväga den ledande halvklassiska approximationen av ett $ 1 $ -dimensionellt problem med Hamiltonian

$$ H (x, p) ~ = ~ \ frac {p ^ 2} {2m} + \ Phi (x), $$

där $ \ Phi $ är en potential. Semiklassiskt ges antalet stater $ N (E) $ under energinivån $ E $ av det område med fasutrymme som är klassiskt tillgängligt, dividerat med Plancks konstanta $ h $,

$ $ N (E) ~ \ approx ~ \ iint_ {H (x, p) \ leq E} \ frac {dx ~ dp} {h}. \ qquad (1) $$

[Här ignorerar vi Maslov-indexet, även känt som metaplektisk korrigering, som t.ex. ger nollpunktsenergi i enkel harmonisk oscillator (SHO) -spektrum.] Låt

$$ V_0 ~: = ~ \ inf_ {x \ in \ mathbb {R} } ~ \ Phi (x) $$

vara infimum för den potentiella energin. Låt

$$ \ ell (V) ~: = ~ \ lambda (\ {x \ in \ mathbb {R} \ mid \ Phi (x) \ leq V \}) $$

vara längden på det klassiskt tillgängliga positionsområdet vid potentiell energinivå $ V $. [Tekniskt sett är längden $ \ ell (V) $ Lebesgue-måttet $ \ lambda $ på förbilden

$$ \ Phi ^ {- 1} (] - \ infty, V]) ~: = ~ \ {x \ in \ mathbb {R} \ mid \ Phi (x) \ leq V \}, $$

som inte nödvändigtvis behöver vara ett anslutet intervall.]

Exempel 1: Om den potentiella $ \ Phi (x) = \ Phi (-x) $ är en jämn funktion och ökar kraftigt monotont för $ x \ geq 0 $, då är den tillgängliga längden $ \ ell (V) = 2 \ Phi ^ {- 1} (V) $ är två gånger den positiva inversa grenen av $ \ Phi $.

Exempel 2: Om potentialen har en hård vägg $ \ Phi (x) = + \ infty $ för $ x<0 $, och ökar kraftigt monotont för $ x \ geq 0 $, då är den tillgängliga längden $ \ ell (V) = \ Phi ^ {- 1} (V) $ den positiva inversa grenen av $ \ Phi $.

Exempel 3: Om den potentiella $ \ Phi (x) $ minskar kraftigt monotont för $ x \ leq0 $ och kraftigt monotont ökar för $ x \ geq 0 $, då är den tillgängliga längden $ \ ell (V) = \ Phi _ {+ } ^ {- 1} (V) - \ Phi _ {-} ^ {- 1} (V) $ är skillnaden mellan de två inversa grenarna av $ \ Phi $.

I exempel 1 och 2, om vi skulle kunna bestämma den tillgängliga längdfunktionen $ \ ell (V) $, skulle vi också kunna generera motsvarande potentiella $ \ Phi (x) $ som OP frågar.

Huvudkravet är att vi kan rekonstruera den tillgängliga längden $ \ ell (V) $ från $ N (E) $ och vice versa. $$ N (E) ~ \ approx ~ \ frac {\ sqrt {2m}} {h} \ int_ {V_0} ^ E \ frac {\ ell (V) ~ dV} {\ sqrt {EV}}, \ qquad (2) $$ $$ \ ell (V) ~ \ approx ~ \ hbar \ sqrt {\ frac {2} {m}} \ frac {d} {dV} \ int_ {V_ {0}} ^ V \ frac {N (E) ~ dE} {\ sqrt {VE}}. \ Qquad (3) $$

[Tecknen $ \ approx $ är för att påminna oss om det halvklassiska approximation (1) vi gjorde. Formlerna kan skrivas i termer av bråkderivat som Jose Garcia påpekar i sitt svar.]

Bevis på ekv. (2):

$$ h ~ N (E) ~ \ stackrel {(1)} {\ approx} ~ 2 \ int_0 ^ {\ sqrt {2m (E-V_0)}} \ left. \ ell (V) \ höger | _ {V = E- \ frac {p ^ 2} {2m}} ~ dp $$ $$ ~ \ stackrel {V = E- \ frac {p ^ 2} {2m}} {=} ~ 2 \ int_ {V_0} ^ E \ frac {\ ell (V) ~ dV} {v} ~ = ~ \ sqrt {2m} \ int_ {V_0} ^ E \ frac {\ ell (V) ~ dV} {\ sqrt {EV}}, $$

eftersom $ dV ~ = ~ - v ~ dp $ med hastigheten $ v ~: = ~ \ frac {p} {m} ~ = ~ \ sqrt {\ frac {2 (EV)} {m}} $.

Bevis på ekv. (3): Observera att

$$ \ int_ {V ^ {\ prime }} ^ V \ frac {dE} {\ sqrt {(VE) (EV ^ {\ prime})}} ~ \ stackrel {E = V \ sin ^ 2 \ theta + V ^ {\ prime} \ cos ^ 2 \ theta} {=} ~ 2 \ int_0 ^ {\ frac {\ pi} {2}} d \ theta ~ = ~ \ pi. \ qquad (4) $$

Sedan

$$ \ frac {h} {\ sqrt {2m}} \ int_ {V_0} ^ V \ frac {N (E) ~ dE} {\ sqrt {VE}} ~ \ stackrel {(2)} { \ approx} ~ \ int_ {V_0} ^ {V} \ frac {dE} {\ sqrt {VE}} \ int_ {V_0} ^ {E} \ frac {\ ell (V ^ {\ prime}) ~ dV ^ {\ prime}} {\ sqrt {EV ^ {\ prime}}} $$$$ ~ \ stackrel {{\ rm Fubini}} {=} ~ \ int_ {V_0} ^ V \ ell (V ^ {\ prime }) ~ dV ^ {\ prime} \ int_ {V ^ {\ prime}} ^ V \ frac {dE} {\ sqrt {(VE) (EV ^ {\ prime})}} ~ \ stackrel {(4) } {=} ~ \ pi \ int_ {V_0} ^ V \ ell (V ^ {\ prime}) ~ dV ^ {\ prime}, \ qquad (5) $$

där vi litar på Fubinis teorem för att ändra ordningen på integrationer. Slutligen skiljer sig tiation wrt. $ V $ på båda sidor av ekv. (5) ger ekv. (3).

en annan fråga .. när vi väl har konstruerat potentialen från $ N (E) $ kan vi bevisa att Hamiltonian $ H = p ^ {2} + V (x) $ kommer att uppfylla Gutzwiller Trace-formeln? . Jag menar om Gutzwillers spårformel och den halvklassiska rekonstruktionen av denna potential är till synes relaterade eller inte :)
en annan kommentar, även om $ N (E) $ alltid är positivt, och om vi tänker på till och med potentiella $ V (x) = V (-x) $, är det åtminstone semiklassiskt möjligt att bevisa att $ V (x) $ kommer att vara positivt om vi vet att $ l (V) $ är positiv ??, tack.
Beräkningar för senare: $ \ frac {1} {\ hbar} \ sqrt {\ frac {m} {2}} \ int_ {V_0} ^ V \ ell (V ^ {\ prime}) ~ dV ^ {\ prime} = \ int_ {V_0} ^ V \ frac {N (E) ~ dE} {\ sqrt {VE}} = 2 \ int_ {V_0} ^ V \! dE ~ N ^ {\ prime} (E) \ sqrt { VE} $ och $ \ frac {1} {\ hbar} \ sqrt {\ frac {m} {2}} \ ell (V) = \ frac {d} {dV} \ int_ {V_ {0}} ^ V \ frac {N (E) ~ dE} {\ sqrt {VE}} = \ int_ {V_0} ^ V \ frac {N ^ {\ prime} (E) ~ dE} {\ sqrt {VE}} $$ = 2N ^ {\ prime} (V_0) \ sqrt {V-V_0} + 2 \ int_ {V_0} ^ V \! DE ~ N ^ {\ prime \ prime} (E) \ sqrt {VE} $ och $ \ frac {h} {\ sqrt {2m}} N (E) = \ int_ {V_0} ^ E \ frac {\ ell (V) ~ dV} {\ sqrt {EV}} = 2 \ ell (V_0) \ sqrt {E-V_0} + 2 \ int_ {V_0} ^ E \! dV ~ \ ell ^ {\ prime} (V) \ sqrt {EV} $.
Beräkningar för senare: $ \ frac {h} {\ sqrt {2m}} N ^ {\ prime} (E) = \ frac {\ ell (V_0)} {\ sqrt {E-V_0}} + \ int_ {V_0 } ^ E \! DV ~ \ frac {\ ell ^ {\ prime} (V)} {\ sqrt {EV}} $.
Beräkningar för senare: $ T (E) = h N ^ {\ prime} (E) $.
Beräkningar för senare: $ \ ell (V_0) = 0 $.
Jose Javier Garcia
2011-10-01 18:56:36 UTC
view on stackexchange narkive permalink

ja, åtminstone för en dimension kan du hindra det omvända problemet

låt vara N (E) trappan för egenvärdet, då i WKB-approximationen ges potentialen invers av

$ V ^ {- 1} (x) = 2 \ sqrt \ pi \ frac {d ^ {1/2}} {dx ^ {1/2}} N (x) $

så att vi kan få det omvända (och därmed potentialen från utvärderingstrappan)

Detta skulle bara fungera för en potential som är smidig och följer $ V (x) = V (-x) $. Sedan bestämmer N (E) potentialen för WKB-order.
detta fungerar också när potentialen är oändlig för x <0 (oändlig vägg vid x = 0) kan du leta efter "bouncer" V = x eller liknande
Kan du ge några referenser Pls?
du kan söka 'Wu och sprung potential' Riemann Hypotes som ett omvänt problem behandlas där :)
Jag tror att om längdfunktionen $ l (x) $ ökar kan du få potential ... i andra fall kan du alltid återspegla funktionen 'l' genom linjen $ y = x $ jag använde detta spektrala problem för att hitta en lämplig Hamilton för Riemann-hypotesen i WKB-fallet
anna v
2015-08-27 08:30:44 UTC
view on stackexchange narkive permalink

Eftersom alla svar är orienterade på den teoretiska sidan, låt mig påminna alla om Balmer-serien, en experimentell observation utrustad med en matematisk serie, som först modellerades med Bohr-modellen av väteatomen, och sedan var hörnstenen i bygga kvantmekanik, som den kom ut från Schrodinger-ekvationen.

Så i denna mening matchade ett givet / uppmätt energispektrum en specifik potential i Schrodinger-ekvationen.

1 ) Finns det en en till en korrespondens mellan potentialen och dess spektrum?

I det historiska exemplet ovan, ja. Jag har inte sett en andra potential som ersätter vätgaspotentialen i modellering.

2) Om svaret på föregående fråga är ja, då ges spektrumet, finns det ett systematiskt sätt att beräkna motsvarande potential

Den historiska exempel är en utbildad försök och felmetod, trots allt var den klassiska potentialen mellan laddningar känd. Det verkar peka på detta: använd motsvarande för generatorn av spektrumets klassiska potential till att börja med. Ett spektrum är en fysisk observation som kommer från specifika atomer / molekyler / ensembler. Så jag skulle ersätta "beräkna" med "hitta". Fysik handlar om att hitta matematiska modeller som passar de observerbara, imo.

Inom en teoretisk ram är "i allmänhet nej" svaret eftersom det finns oändliga möjliga lösningar. Var vi verkligen lyckliga att ha upptäckt kvantmekanik, om sannolikheten att hitta den potential som passar ett spektrum är så liten? IMO nej, fysikerna använde sin fysikbakgrund för att hitta en bra teoretisk modell som passar observationerna (Balmer-serien) och använder sin klassiska kunskap om potentialer som en gräns för oändligheten av lösningar. Det är en liknande logisk process som att använda gränsvillkor för att minska oändligheten av lösningar från differentialekvationerna till en som passar data.

hbp
2015-10-12 23:59:51 UTC
view on stackexchange narkive permalink

Jag redogör något för lösningen genom qmechanic och ger två exempel. Lösningen är nära relaterad till Landaus metod för periodinversion, se Mekanik sek. 12.

Kvantantalet kan beräknas ungefär som \ begin {align} (n + \ delta) \, 2 \ pi \ hbar& = \ oint p \, dq \\ & = \ oint \ sqrt {2 \, m \, [E - V (q)]} \, dq \\ & = 4 \ int_0 ^ {r_ \ max} \ sqrt {2 \, m \, [E - V (q)]} \, dq, \ tag {1} \ end {align} där $ \ delta $ är ett tal mellan $ 0 $ och $ 1 $ (dvs. $ 1/2 $) och $ \ hbar $ är Planck-konstanten. Den integrerade $ \ oint $ betyder "gå dit och komma tillbaka". I det sista steget har vi antagit att $ V (q) $ är centrosymmetrisk och $ V (r_ \ max) = E $. Differentierar (1) med avseende på $ E $, vi får \ begin {align} 2 \ pi \ hbar \, \ frac {\ partial n} {\ partial E} & = \ int_0 ^ {r_ \ max} \ frac { \ sqrt {8 m} \, dq} {\ sqrt {E - V (q)}}, \ tag {1} \ end {align} Multiplicera detta med $ 1 / \ sqrt {2 \, m \, (\ alpha - E)} $ och integreras över $ E $, vi får \ begin {align} 2 \ pi \ hbar \, \ int_0 ^ \ alpha \ frac {\ partial n} {\ partial E} \ frac {dE} {\ sqrt {2 \, m \, (\ alpha - E)}} & = \ int_0 ^ {r_ \ max} 2 \, dq \ int_ {V (q)} ^ \ alpha \ frac {dE} {\ sqrt { \ alpha - E} {\ sqrt {E - V (q)}}} \\ & = 2 \, \ pi \, r_ \ max. \ end {align} Nu sedan $ V (r_ \ max) = \ alpha $, den potentiella $ V (r) $ löses via den inversa funktionen som \ begin {align} \ hbar \, \ int_0 ^ \ alpha \ frac {\ partial n} {\ partial E} \ frac {dE} {\ sqrt {2 \, m \, (\ alpha - E)}} & = V ^ {- 1} (\ alpha), \ end {align} där den inversa funktionen tas för $ r > 0 $ -grenen.

Exempel 1

För den harmoniska oscillatorn, $ E_n = \ hbar \, \ omega (n + 1/2) $, sedan $ \ partial n / \ partial E = (\ hbar \, \ omega) ^ {- 1} $ och \ börja {align} \ hbar \, \ int_0 ^ \ alpha \ frac {\ partial n} {\ partial E} \ frac {dE} {\ sqrt {2 \, m \, (\ alpha - E)}} & = \ sqrt {\ frac {2 \, \ alpha} {m \, \ omega ^ 2}} = r. \ end {align} vilket innebär att potentialen är $ V (r) = \ alpha = \ frac {1} { 2} m \, \ omega ^ 2 \, r ^ 2 $, vilket är korrekt.

Exempel 2

Om $ E_n = (\ hbar \ pi n) ^ 2 / (2 m L ^ 2) $, då $ \ partial n / \ partial E = L / (\ hbar \, \ pi) \ sqrt {m / (2 \, E)} $ och \ börja {align} \ hbar \, \ int_0 ^ \ alpha \ frac {\ partial n} {\ partial E} \ frac {dE} {\ sqrt {2 \, m \, (\ alpha - E)}} & = \ frac {L} {2 \, \ pi} \, \ int_0 ^ \ alpha \ frac {dE} {\ sqrt {E \, (\ alpha - E)} } \\ & = \ frac {L} {2} = r. \ End {align} Detta betyder att den inversa funktionen $ V ^ {- 1} (\ alpha) $ är en konstant $ L / 2 $ oavsett värdet på $ \ alpha $ (även om $ \ alpha $ växer till $ + \ infty $). Med andra ord växer $ V (r) $ brant till oändlighet vid $ r = L / 2 $. Eftersom vi antar att potentialen är symmetrisk växer $ V (r) $ också till oändlighet vid $ r = -L / 2 $. Detta innebär att potentialen har två symmetriska oändliga hårda väggar vid $ r = \ pm L / 2 $. Detta är fallet med en partikel i en låda med storlek $ L $. Tack till Kyle-Kanos för att påpeka detta.

Är inte den potentiella $ V (\ hat {x}) = \ frac12m \ omega ^ 2 \ hat {x} ^ 2 $ för QHO?Hur får du också potentialen för partikel-i-en-lådan från $ L / 2 = r $?
Tyvärr, jag saknade $ r ^ 2 $ i exempel 1. Tack för att du påpekade detta.Exempel 2 är $ V ^ {- 1} (\ alpha) $ en konstant, vilket betyder att det är en horisontell linje när vi plottar $ r $ mot $ V $ och sedan när vi plottar $ V $ mot $ r $,vi får en vertikal linje.Kom också ihåg att vi antar att $ V $ är centrosymmetrisk, så spegelpotentiallinjen finns också vid $ r = -L / 2 $.Detta är exakt den potentiella formen på partikeln i en låda, med lådstorleken $ L $.
porphyrin
2016-07-28 12:52:01 UTC
view on stackexchange narkive permalink

Som ett tillägg till svaret av @Jose Javier Garcia och i andan av svaret av @anna v, i molekylär spektroskopi, beskrivs ofta anharmoniska potentialer ungefär av Morse Potential, men detta är otillräckligt i de flesta fall och dåRydberg-Klein-Rees (RKR) numeriska metod används mycket allmänt för att erhålla den potentiella energiprofilen.Denna metod baseras på den semiklassiska WKB-metoden.(Se Hirst, "Potentiella energiytor; Molekylär struktur och reaktionsdynamik" och artiklar på Wikipedia).



Denna fråga och svar översattes automatiskt från det engelska språket.Det ursprungliga innehållet finns tillgängligt på stackexchange, vilket vi tackar för cc by-sa 3.0-licensen som det distribueras under.
Loading...